How Can the Given Definite Integral Identity Be Proven for Any Natural Number n?

Dan have discussed the identity \[\int_{0}^{1}\frac{x^{n-1}+x^{n-\frac{1}{2}}-2x^{2n-1}}{1-x}dx = 2\ln2,\] and Theia has provided a clever solution using derivatives. Dan notes that this implies that n doesn't have to be a natural number, and Theia agrees that F'(n) = 0 is not a crucial requirement for the identity. They both continue to ponder on the question of what is required for n. In summary, the discussion centers around the identity and its implications for the value of n.
  • #1
lfdahl
Gold Member
MHB
749
0
Show, that the identity

\[\int_{0}^{1}\frac{x^{n-1}+x^{n-\frac{1}{2}}-2x^{2n-1}}{1-x}dx = 2\ln2\]

- holds for any natural number $n$.
 
Mathematics news on Phys.org
  • #2
Here´s the suggested solution:

Rewriting the integrand:

\[\frac{x^{n-1}+x^{n-1/2}-2x^{2n-1}}{1-x}=\frac{x^{n-1}(1+\sqrt{x}-2x^n)}{1-x}=\frac{x^{n-1}((1-\sqrt{x})(1+2\sqrt{x})+2x-2x^n)}{1-x}\]

\[=\frac{x^{n-1}(1+2\sqrt{x})}{1+\sqrt{x}}+\frac{2x^n(1-x^{n-1})}{1-x}\\\\=x^{n-1}+x^{n-1}\frac{\sqrt{x}}{1+\sqrt{x}}+2x^n\:\frac{(1-x)(1+x+...+x^{n-3}+x^{n-2})}{1-x} \\\\=2x^{n-1}-\frac{x^{n-1}}{1+\sqrt{x}}+2\left ( x^n+x^{n+1}+...+x^{2n-2} \right ) \\\\=2\left ( x^{n-1}+x^n+...+x^{2n-2} \right )-\frac{x^{n-1}}{1+\sqrt{x}}\]

Integrating yields:
\[I = \int_{0}^{1} \left ( 2\left ( x^{n-1}+x^n+...+x^{2n-2} \right )-\frac{x^{n-1}}{1+\sqrt{x}} \right )dx = 2\sum_{j=n}^{2n-1}\frac{1}{j}-\int_{0}^{1}\frac{x^{n-1}}{1+\sqrt{x}}dx\]
The last term can be rewritten as ($x = u^2$):
\[\int_{0}^{1}\frac{x^{n-1}}{1+\sqrt{x}}dx = 2\int_{0}^{1}\frac{u^{2n-1}}{1+u }du\]

Rewriting the integrand using the geometric series:

\[\frac{u^{2n-1}}{1+u} = u^{2n-1}\sum_{i=0}^{\infty}(-u)^i = u^{2n-1}-u^{2n}+u^{2n+1}-... \]

The right hand side is thus obtained by cutting off the first $2n-1$ terms of an alternating geometric series:

\[\frac{1}{1+u}=1-u+u^2-u^3+...+u^{2n-2}-u^{2n-1}+u^{2n}-u^{2n+1}... \\\\=1-u+u^2-u^3+...+u^{2n-2}- \left ( u^{2n-1}-u^{2n}+u^{2n+1}... \right )\]

Thus, the integral can be written:

\[2\int_{0}^{1}\frac{u^{2n-1}}{1+u }du = 2\int_{0}^{1}\left ( 1-u+u^2-u^3+...+u^{2n-2}-\frac{1}{1+u} \right )du \\\\=2\left ( \sum_{j=1}^{2n-1}\frac{(-1)^{j-1}}{j}-\ln 2 \right )\]

Our total integral now has the form:

\[I =2\sum_{j=n}^{2n-1}\frac{1}{j}- 2\left ( \sum_{j=1}^{2n-1}\frac{(-1)^{j-1}}{j}-\ln 2 \right )\\\\ =2\sum_{j=n}^{2n-1}\frac{1}{j}-2\left ( \sum_{j=1}^{2n-1}\frac{1}{j} - 2\sum_{j=1}^{n-1}\frac{1}{2j}\right )+2\ln 2 \\\\=2\left ( \sum_{j=1}^{2n-1}\frac{1}{j}-\left ( \sum_{j=1}^{2n-1}\frac{1}{j}\right ) \right )+2\ln 2 \\\\= 2 \ln 2.\]
- and we´re done.
 
  • #3
I'm sorry about poking a bit older threat, but I found this interesting, so here is my attempt:

Let's define

\(\displaystyle F(n) = \int_0^1 \frac{x^{n-1} + x^{n-1/2} - 2x^{2n-1}}{1-x}\mathrm{d}x \qquad \forall \ n \in \mathbb{N}.\)

One wants to show that the value of \(\displaystyle F(n)\) doesn't change, i.e. it's constant. To show this, one needs to show that \(\displaystyle F'(n) = 0\), where \(\displaystyle '\) denotes the derivative with respect to \(\displaystyle n\).

Let's calculate the derivative by applying Leibniz's rule of derivating under integral sign. One obtains

\(\displaystyle \begin{align*}F'(n) &= \frac{\mathrm{d}F}{\mathrm{d}n} = \int_0^1 \frac{\partial}{\partial n} \left( \frac{x^{n-1} + x^{n-1/2} - 2x^{2n-1}}{1-x} \right) \mathrm{d}x \\
& = \int_0^1 \frac{x^{n-1} + x^{n-1/2} - 4x^{2n-1}}{1-x} \ln x \ \mathrm{d}x \ .\end{align*}\)

Next one uses the geometric serie to rewrite denominator (\(\displaystyle 0 \le x < 1\)):

\(\displaystyle \frac{1}{1-x} = \sum_{v = 0}^{\infty} x^v = 1 + x + x^2 + \cdots . \)

Using this, the derivative of \(\displaystyle F\) can be written as

\(\displaystyle \begin{align*}F'(n) &= \int_0^1 \left( x^{n-1} + x^{n-1/2} - 2x^{2n-1} \right) \ln x \sum_{v = 0}^{\infty} x^v \ \mathrm{d}x \\
&= \int_0^1 \sum_{v = 0}^{\infty} x^{v+n-1} \ln x \ \mathrm{d}x + \int_0^1 \sum_{v = 0}^{\infty} x^{v+n-1/2} \ln x \ \mathrm{d}x \\
&\phantom{=-} -4 \int_0^1 \sum_{v = 0}^{\infty} x^{v+2n-1} \ln x \ \mathrm{d}x \ .\end{align*}\)

Now one can change the order of summation and integration (by e.g. Fubini's theorem) and one obtains

\(\displaystyle \begin{align*}F'(n) &= \sum_{v = 0}^{\infty} \int_0^1 x^{v+n-1} \ln x \ \mathrm{d}x + \sum_{v = 0}^{\infty} \int_0^1 x^{v+n-1/2} \ln x \ \mathrm{d}x \\
&\phantom{=} -4 \sum_{v = 0}^{\infty} \int_0^1 x^{v+2n-1} \ln x \ \mathrm{d}x \ .\end{align*}\)

Now integrations are trivial and one can write

\(\displaystyle \begin{align*}F'(n) &= \sum_{v = 0}^{\infty} {\Big/}_{\!\!\!0}^{1} x^{v+n-1} \ln x + \sum_{v = 0}^{\infty} {\Big/}_{\!\!\!0}^{1} x^{v+n-1/2} \ln x \\
&\phantom{=} -4 \sum_{v = 0}^{\infty} {\Big/}_{\!\!\!0}^{1} x^{v+2n-1} \ln x \\
&= -\sum_{v=0}^{\infty} \frac{1}{\left( v+n \right) ^2} - \sum_{v=0}^{\infty} \frac{1}{\left( v+n+\tfrac{1}{2} \right) ^2} + 4 \sum_{v=0}^{\infty} \frac{1}{\left( v+2n \right) ^2} . \end{align*}\)

In terms of polygamma function \(\displaystyle \psi ^{(m)}\) this can be written

\(\displaystyle F'(n) = 4\psi ^{(1)}(2n) - \psi ^{(1)}(n + \tfrac{1}{2}) - \psi ^{(1)}(n) \ . \)

By using the multiplication formula one can rewrite the \(\displaystyle 4\psi ^{(1)}(2n)\):

\(\displaystyle 4\psi ^{(1)}(2n) = \psi ^{(1)}(n + \tfrac{1}{2}) + \psi ^{(1)}(n) \ , \)

which gives the result \(\displaystyle F'(n) = 0 \ n \in \mathbb{N}\), and hence \(\displaystyle F(n) = \textrm{constant}.\)

The value of \(\displaystyle F(n)\) one can evaluate by substituting e.g. \(\displaystyle n = 1\), which gives

\(\displaystyle \begin{align*} F(n) &= F(1) = \int_0^1 \frac{x^0 + x^{1/2} - 2x^1}{1-x}\mathrm{d}x \\
&= {\Big/}_{\!\!\!0}^{1} \left( 2x - 2\sqrt{x} + 2 \ln \left( \sqrt{x} + 1 \right) \right) \\
&= 2 \ln 2 , \end{align*}\)

which is what one wanted to show. QED.
 
  • #4
Thankyou, Theia, for a clever solution! :cool:
 
  • #5
Theia said:
I'm sorry about poking a bit older threat, but I found this interesting, so here is my attempt:

Let's define

\(\displaystyle F(n) = \int_0^1 \frac{x^{n-1} + x^{n-1/2} - 2x^{2n-1}}{1-x}\mathrm{d}x \qquad \forall \ n \in \mathbb{N}.\)

One wants to show that the value of \(\displaystyle F(n)\) doesn't change, i.e. it's constant. To show this, one needs to show that \(\displaystyle F'(n) = 0\), where \(\displaystyle '\) denotes the derivative with respect to \(\displaystyle n\).

Let's calculate the derivative by applying Leibniz's rule of derivating under integral sign. One obtains

\(\displaystyle \begin{align*}F'(n) &= \frac{\mathrm{d}F}{\mathrm{d}n} = \int_0^1 \frac{\partial}{\partial n} \left( \frac{x^{n-1} + x^{n-1/2} - 2x^{2n-1}}{1-x} \right) \mathrm{d}x \\
& = \int_0^1 \frac{x^{n-1} + x^{n-1/2} - 4x^{2n-1}}{1-x} \ln x \ \mathrm{d}x \ .\end{align*}\)

Next one uses the geometric serie to rewrite denominator (\(\displaystyle 0 \le x < 1\)):

\(\displaystyle \frac{1}{1-x} = \sum_{v = 0}^{\infty} x^v = 1 + x + x^2 + \cdots . \)

Using this, the derivative of \(\displaystyle F\) can be written as

\(\displaystyle \begin{align*}F'(n) &= \int_0^1 \left( x^{n-1} + x^{n-1/2} - 2x^{2n-1} \right) \ln x \sum_{v = 0}^{\infty} x^v \ \mathrm{d}x \\
&= \int_0^1 \sum_{v = 0}^{\infty} x^{v+n-1} \ln x \ \mathrm{d}x + \int_0^1 \sum_{v = 0}^{\infty} x^{v+n-1/2} \ln x \ \mathrm{d}x \\
&\phantom{=-} -4 \int_0^1 \sum_{v = 0}^{\infty} x^{v+2n-1} \ln x \ \mathrm{d}x \ .\end{align*}\)

Now one can change the order of summation and integration (by e.g. Fubini's theorem) and one obtains

\(\displaystyle \begin{align*}F'(n) &= \sum_{v = 0}^{\infty} \int_0^1 x^{v+n-1} \ln x \ \mathrm{d}x + \sum_{v = 0}^{\infty} \int_0^1 x^{v+n-1/2} \ln x \ \mathrm{d}x \\
&\phantom{=} -4 \sum_{v = 0}^{\infty} \int_0^1 x^{v+2n-1} \ln x \ \mathrm{d}x \ .\end{align*}\)

Now integrations are trivial and one can write

\(\displaystyle \begin{align*}F'(n) &= \sum_{v = 0}^{\infty} {\Big/}_{\!\!\!0}^{1} x^{v+n-1} \ln x + \sum_{v = 0}^{\infty} {\Big/}_{\!\!\!0}^{1} x^{v+n-1/2} \ln x \\
&\phantom{=} -4 \sum_{v = 0}^{\infty} {\Big/}_{\!\!\!0}^{1} x^{v+2n-1} \ln x \\
&= -\sum_{v=0}^{\infty} \frac{1}{\left( v+n \right) ^2} - \sum_{v=0}^{\infty} \frac{1}{\left( v+n+\tfrac{1}{2} \right) ^2} + 4 \sum_{v=0}^{\infty} \frac{1}{\left( v+2n \right) ^2} . \end{align*}\)

In terms of polygamma function \(\displaystyle \psi ^{(m)}\) this can be written

\(\displaystyle F'(n) = 4\psi ^{(1)}(2n) - \psi ^{(1)}(n + \tfrac{1}{2}) - \psi ^{(1)}(n) \ . \)

By using the multiplication formula one can rewrite the \(\displaystyle 4\psi ^{(1)}(2n)\):

\(\displaystyle 4\psi ^{(1)}(2n) = \psi ^{(1)}(n + \tfrac{1}{2}) + \psi ^{(1)}(n) \ , \)

which gives the result \(\displaystyle F'(n) = 0 \ n \in \mathbb{N}\), and hence \(\displaystyle F(n) = \textrm{constant}.\)

The value of \(\displaystyle F(n)\) one can evaluate by substituting e.g. \(\displaystyle n = 1\), which gives

\(\displaystyle \begin{align*} F(n) &= F(1) = \int_0^1 \frac{x^0 + x^{1/2} - 2x^1}{1-x}\mathrm{d}x \\
&= {\Big/}_{\!\!\!0}^{1} \left( 2x - 2\sqrt{x} + 2 \ln \left( \sqrt{x} + 1 \right) \right) \\
&= 2 \ln 2 , \end{align*}\)

which is what one wanted to show. QED.
Interesting. I note that Theia's solution uses a derivative of n, which implies that n doesn't have to be natural number! (I double-checked this on W|A.)

-Dan
 
  • #6
Yes, \(\displaystyle n \in \mathbb{N}\) is not a crucial requirement for \(\displaystyle F'(n) = 0\). But I haven't yet found a proof that shows what is required for \(\displaystyle n\).(Wondering)
 
  • #7
Theia said:
I'm sorry about poking a bit older threat, but I found this interesting, so here is my attempt:

Let's define

\(\displaystyle F(n) = \int_0^1 \frac{x^{n-1} + x^{n-1/2} - 2x^{2n-1}}{1-x}\mathrm{d}x \qquad \forall \ n \in \mathbb{N}.\)

One wants to show that the value of \(\displaystyle F(n)\) doesn't change, i.e. it's constant. To show this, one needs to show that \(\displaystyle F'(n) = 0\), where \(\displaystyle '\) denotes the derivative with respect to \(\displaystyle n\).

Let's calculate the derivative by applying Leibniz's rule of derivating under integral sign. One obtains

\(\displaystyle \begin{align*}F'(n) &= \frac{\mathrm{d}F}{\mathrm{d}n} = \int_0^1 \frac{\partial}{\partial n} \left( \frac{x^{n-1} + x^{n-1/2} - 2x^{2n-1}}{1-x} \right) \mathrm{d}x \\
& = \int_0^1 \frac{x^{n-1} + x^{n-1/2} - 4x^{2n-1}}{1-x} \ln x \ \mathrm{d}x \ .\end{align*}\)

Next one uses the geometric serie to rewrite denominator (\(\displaystyle 0 \le x < 1\)):

\(\displaystyle \frac{1}{1-x} = \sum_{v = 0}^{\infty} x^v = 1 + x + x^2 + \cdots . \)

Using this, the derivative of \(\displaystyle F\) can be written as

\(\displaystyle \begin{align*}F'(n) &= \int_0^1 \left( x^{n-1} + x^{n-1/2} - 2x^{2n-1} \right) \ln x \sum_{v = 0}^{\infty} x^v \ \mathrm{d}x \\
&= \int_0^1 \sum_{v = 0}^{\infty} x^{v+n-1} \ln x \ \mathrm{d}x + \int_0^1 \sum_{v = 0}^{\infty} x^{v+n-1/2} \ln x \ \mathrm{d}x \\
&\phantom{=-} -4 \int_0^1 \sum_{v = 0}^{\infty} x^{v+2n-1} \ln x \ \mathrm{d}x \ .\end{align*}\)

Now one can change the order of summation and integration (by e.g. Fubini's theorem) and one obtains

\(\displaystyle \begin{align*}F'(n) &= \sum_{v = 0}^{\infty} \int_0^1 x^{v+n-1} \ln x \ \mathrm{d}x + \sum_{v = 0}^{\infty} \int_0^1 x^{v+n-1/2} \ln x \ \mathrm{d}x \\
&\phantom{=} -4 \sum_{v = 0}^{\infty} \int_0^1 x^{v+2n-1} \ln x \ \mathrm{d}x \ .\end{align*}\)

Now integrations are trivial and one can write

\(\displaystyle \begin{align*}F'(n) &= \sum_{v = 0}^{\infty} {\Big/}_{\!\!\!0}^{1} x^{v+n-1} \ln x + \sum_{v = 0}^{\infty} {\Big/}_{\!\!\!0}^{1} x^{v+n-1/2} \ln x \\
&\phantom{=} -4 \sum_{v = 0}^{\infty} {\Big/}_{\!\!\!0}^{1} x^{v+2n-1} \ln x \\
&= -\sum_{v=0}^{\infty} \frac{1}{\left( v+n \right) ^2} - \sum_{v=0}^{\infty} \frac{1}{\left( v+n+\tfrac{1}{2} \right) ^2} + 4 \sum_{v=0}^{\infty} \frac{1}{\left( v+2n \right) ^2} . \end{align*}\)

In terms of polygamma function \(\displaystyle \psi ^{(m)}\) this can be written

\(\displaystyle F'(n) = 4\psi ^{(1)}(2n) - \psi ^{(1)}(n + \tfrac{1}{2}) - \psi ^{(1)}(n) \ . \)

By using the multiplication formula one can rewrite the \(\displaystyle 4\psi ^{(1)}(2n)\):

\(\displaystyle 4\psi ^{(1)}(2n) = \psi ^{(1)}(n + \tfrac{1}{2}) + \psi ^{(1)}(n) \ , \)

which gives the result \(\displaystyle F'(n) = 0 \ n \in \mathbb{N}\), and hence \(\displaystyle F(n) = \textrm{constant}.\)

The value of \(\displaystyle F(n)\) one can evaluate by substituting e.g. \(\displaystyle n = 1\), which gives

\(\displaystyle \begin{align*} F(n) &= F(1) = \int_0^1 \frac{x^0 + x^{1/2} - 2x^1}{1-x}\mathrm{d}x \\
&= {\Big/}_{\!\!\!0}^{1} \left( 2x - 2\sqrt{x} + 2 \ln \left( \sqrt{x} + 1 \right) \right) \\
&= 2 \ln 2 , \end{align*}\)

which is what one wanted to show. QED.
It really is an interesting question.

I've been thinking about it this evening, and wonder if what's below makes sense.
I was thinking about an alternative way to show that the derivative vanishes in your solution. For $\alpha \ge 1$, let $\displaystyle f(\alpha) = \int_0^1 \frac{x^{\alpha-1}+x^{\alpha-\frac{1}{2}}-2x^{2\alpha-1}}{1-x}\,\mathrm{dx}.$ Then $\displaystyle f'(\alpha) = \int_0^1 \frac{x^{\alpha-1}+x^{\alpha-\frac{1}{2}}-4x^{2\alpha-1}}{1-x} \log(x)\,\mathrm{dx}.$

For any $x \in (0, 1)$ we have $(1-\frac{1}{x}) \leqslant \log(x) \leqslant x-1. $ Applying this to the integral, we have

$\displaystyle \int_0^1 \frac{x^{\alpha-1}+x^{\alpha-\frac{1}{2}}-4x^{2\alpha-1}}{1-x} (1-\frac{1}{x})\,\mathrm{dx} \leqslant f'(\alpha) \leqslant \int_0^1 \frac{(x^{\alpha-1}+x^{\alpha-\frac{1}{2}}-4x^{2\alpha-1}(x-1)}{1-x} \,\mathrm{dx}$

$\implies \displaystyle \int_0^1 {4x^{2\alpha-2}-x^{\alpha-\frac{3}{2}}-x^{\alpha-2}
} \,\mathrm{dx} \leqslant f'(\alpha)\leqslant \int_0^1(4x^{2\alpha-1} -x^{\alpha-\frac{1}{2}}-x^{\alpha-1}) \,\mathrm{dx} $

Hence $ \displaystyle \frac{1}{-2\alpha^2+3\alpha-1} \leqslant f'(\alpha) \leqslant \frac{1}{2\alpha^2+\alpha}$. Solving these with the assumption $\alpha \geqslant 1$ we've:

$ \displaystyle \frac{1}{-2\alpha^2+3\alpha-1} \leqslant f'(\alpha) \implies f'(\alpha) \in [0,\infty); $ and $ \displaystyle f'(\alpha) \leqslant \frac{1}{2\alpha^2+\alpha} \implies f'(\alpha) \in (-\infty,0].$

So $f'(\alpha) \in (-\infty, 0] \cap [0, \infty) = \left\{0\right\}$. So $f'(\alpha) \in \left\{0\right\}$ so $f'(\alpha) = 0$ for all $\alpha \ge 1$. Thus $f$ is constant.

Not sure if I've made a mistake somewhere. But the approach should work nonetheless I feel.
 
  • #8
Well... It has been a while, isn't it? (Wave)

Getting back to this problem, as it was very interesting. Now I have following solution for it:

Let's define

\[ g(a) = \int_{0} ^{1} \frac{x^{a}}{1 - x} \mathrm{d}x = \sum_{v=0} ^{\infty} \int_{0} ^{1} x^{a+v}\mathrm{d}x = \sum_{v=0} ^{\infty} \frac{1}{a+v+1}. \]

Now the integral in question can be written as

\[ F(n) = g(n-1) + g \left( n-\frac{1}{2} \right) - 2g(2n-1) = \sum_{v=0} ^{\infty} \left( \frac{1}{v + n} + \frac{1}{v + \left( n + \frac{1}{2} \right)} - \frac{2}{v + 2n} \right). \]

By noticing that \( a + v \ne -1 \) one arrives to a condition \( n \ne 0, -\frac{1}{2}, -1, -\frac{3}{2}, \ldots \).

Then by using identities of digamma function one directly obtains

\[ F(n) = 2\psi (2n+1) - \psi (n+1) - \psi \left(n + \frac{3}{2}\right) + \frac{1}{n} + \frac{1}{n + \frac{1}{2}} - \frac{2}{2n} = 2\ln 2.\]

Ok, but what if \( n = 0, -\frac{1}{2} , -1, -\frac{3}{2}, \ldots \)?

Let's write \( n = -\frac{p}{2} \) where \(p = 0, 1, \ldots \). After that one can substitute \( x = \frac{1}{q^{2}} \) and one obtains

\[ F\left( n = -\frac{p}{2} \right) = 2\int_{1} ^{\infty} \frac{q^{p} + q^{p+1} - 2q^{2p+1}}{q^{2} - 1} \mathrm{d}q \].

This is clearly divergent.

Hence it looks like the value of the integral equals to \( 2\ln 2\) as long as \(n \ne 0, -\frac{1}{2}, -1, -\frac{3}{2}, \ldots \).

Any thoughts on this?
 
  • #9
Hello,Theia

Unfortunately, I am not familiar with the digamma function, so I am not competent
to answer and comment on your interesting solution in a proper manner. :unsure:

I do hope, that someone else in this forum can be of help here ...
 

Related to How Can the Given Definite Integral Identity Be Proven for Any Natural Number n?

1. What is a definite integral challenge?

A definite integral challenge is a mathematical problem that involves finding the area under a curve between two specified points on the x-axis. It is a common type of problem in calculus and is used to calculate values such as displacement, velocity, and acceleration.

2. How do you solve a definite integral challenge?

To solve a definite integral challenge, you must first determine the function that represents the curve and then use the fundamental theorem of calculus to evaluate the integral. This involves finding the antiderivative of the function and plugging in the upper and lower limits of integration.

3. What are some common strategies for solving definite integral challenges?

Some common strategies for solving definite integral challenges include using the power rule, substitution, and integration by parts. It is also important to understand the properties of integrals, such as linearity and the addition and subtraction rules.

4. What are some real-world applications of definite integrals?

Definite integrals have many real-world applications, such as calculating the area under a velocity-time graph to determine displacement, finding the volume of irregularly shaped objects, and calculating the work done by a variable force.

5. How can I improve my skills in solving definite integral challenges?

To improve your skills in solving definite integral challenges, it is important to practice regularly and familiarize yourself with different types of problems. You can also seek help from a tutor or join a study group to gain a better understanding of the concepts and techniques involved.

Similar threads

Replies
4
Views
509
Replies
12
Views
965
Replies
14
Views
1K
Replies
6
Views
954
Replies
18
Views
2K
Replies
2
Views
1K
  • General Math
Replies
4
Views
738
Replies
3
Views
1K
  • Calculus and Beyond Homework Help
Replies
3
Views
1K
Back
Top